0 Daumen
251 Aufrufe

Ich muss mittels vollständiger Induktion beweisen, dass für die Folge \(a_{n+1}:=\sqrt{1+a_n}\) mit \(a_1=1\) für alle \(n\in\mathbb{N}\) folgendes gilt: \(a_n<2\).

Induktionsanfang:
$$a_1=\sqrt{1+a_0}=\sqrt{1+0}=1<2$$
Induktionsvoraussetzung:
$$\exists n\in\mathbb{N}:\sqrt{1+a_{n-1}}<2$$
Induktionsbehauptung:
$$a_{n+1}=\sqrt{1+a_n}<2$$
Induktionsschritt:

$$2>\sqrt{1+a_{n}}=\sqrt{1+\underbrace{\sqrt{1+a_{n-1}}}_{IV}}\\ =\lim_{t\to2}{\sqrt{1+t}}=\sqrt{3},\quad t=\sqrt{1+a_{n-1}}$$

Da ich mich noch nicht wirklich mit dem Beweisprinzip auskenne, weiß ich nicht ob mein Ansatz richtig ist. Schließlich habe ich \(t=\sqrt{1+a_{n-1}}\) substituiert aber darf man das und ist der Beweis vollständig und richtig?


Vielen Dank im Voraus : )

Avatar von

1 Antwort

0 Daumen

Hallo

1. das fängt mit a1 an nicht mit a0, deshalb ist dein Anfang schlecht.

a und du musst nicht so kompliziert denken

Indvors: an<2 daraus folgt an+1=√(1+an)<√(1+2)=√3<2 fertig

Gruß lul

Avatar von 107 k 🚀

Hallo,

danke für den Tipp ; )

Gruß FyZe18

Ein anderes Problem?

Stell deine Frage

Willkommen bei der Mathelounge! Stell deine Frage einfach und kostenlos

x
Made by a lovely community